Answer:
24 minutes
The answer is 24 I've had that question before
What is the awnser to 6 x 17?
the answer to 6×7 is equal to 102
Which graph matches X<8 below?
Answer:
Option 1
Step-by-step explanation:
I'm working on a question on data structures and algorithms.
Prove that the algorithm given below is correct using the loop invariant theorem. Also, state the choice of loop invariant.
The algorithm is as follows:
(1) initialize j = 0.
(2) While j ≤ m, do:
i. Increment j.
ii. If j divides m, output j.
The algorithm is correct and can be proven using the loop invariant theorem. The loop invariant for this algorithm is that at the start of each iteration of the loop, the value of j is a divisor of m.
To prove the correctness of the algorithm using the loop invariant theorem, we need to establish three properties: initialization, maintenance, and termination.
Initialization: Before the loop starts, j is initialized to 0. At this point, the loop invariant holds because 0 is a divisor of any positive integer m.
Maintenance: Assuming the loop invariant holds at the start of an iteration, we need to show that it holds after the iteration. In this algorithm, j is incremented by 1 in each iteration. Since j starts as a divisor of m, adding 1 to j does not change its divisibility property. Therefore, the loop invariant is maintained.
Termination: The loop terminates when j becomes greater than m. At this point, the loop invariant still holds because j is not a divisor of m. Thus, the loop invariant is maintained throughout the entire execution of the algorithm.
Since the initialization, maintenance, and termination properties hold, we can conclude that the algorithm is correct. The loop invariant, in this case, is that at the start of each iteration, the value of j is a divisor of m.
Learn more about algorithm here:
https://brainly.com/question/21172316
#SPJ11
What is 6/7 x 14 pls help me
Answer:
\( \sf \: \frac{6}{7} \times 14 = 12\)
Step-by-step explanation:
Given problem,
\( \sf \rightarrow \: \frac{6}{7} \times 14\)
Let's solve the problem,
\( \sf \rightarrow \: \frac{6}{7} \times 14\)
\( \sf \rightarrow \frac{6}{\cancel{ \: 7 }} \times \cancel{14}\)
\( \sf \rightarrow \: 6 \times 2\)
\( \sf \rightarrow \: 12\)
Hence, the answer is 12.
WELP! I NEED THE SOLUTION TO THIS EXPRESSION! PLS HELP!
0.5x = 0.3
Answer:
0.5x = 0.3
1x/2 = 0.3
x/2 = 0.3
2 * x/2 = 2 . 0.3
x = 2 . 0.3
x = 0.6
Step-by-step explanation:
I think this is right!
Chi needs to simplify the expression below. (1.25 -0.4)-7+4x3 Which operation should she perform first?
I need an answer quickly
Answer:
She should first perform the operation in the parentheses, you can reference the order of the operations based on PEMDAS.
Step-by-step explanation:
1. parentheses operations
2. multiply 4 x 3
3. add the value you get from the parentheses with -7
4. with that value add it to the product of 4 and 3
Hope that helped! :)
Answer:
Subtraction
Explanation:-
\(( 1.25 -0.4) \div7+ 4 \times 3\)
Using BODMAS Rule:-
BracketsOrdersDivisionMultiplicationAdditionSubtractionIn bracket, the operation subtraction should be performed first .
The domestic violence study conducted in 1984 by Sherman and Berk had an ethical concern in that: O They financially profited from the research. O They did not adhere to special protections for vulnerable populations. O They potentially withheld a beneficial treatment. O They deceived their subjects.
In the 1984 domestic violence study conducted by Sherman and Berk, the ethical concern was that they potentially withheld a beneficial treatment.
The domestic violence study conducted by Sherman and Berk in 1984 raised an ethical concern in that they financially profited from the research. This raises the question of whether their motives were purely altruistic or whether they were driven by financial gain. Additionally, the study did not adhere to special protections for vulnerable populations such as women and children who may have been victims of domestic violence. This raises concerns about the validity and generalizability of the study's findings. Furthermore, the study potentially withheld a beneficial treatment, which raises questions about the ethical responsibility of researchers to ensure that their subjects receive the best possible care. Finally, there are also concerns that the researchers may have deceived their subjects, which raises questions about the integrity and transparency of the research process. In conclusion, the ethical concerns raised by this study highlight the need for researchers to carefully consider the impact of their research on vulnerable populations and to ensure that they adhere to the highest ethical standards.
Visit here to learn more about ethical concerns:
brainly.com/question/28318353
#SPJ11
Lowell has a credit card that uses the adjusted balance method. For the first
10 days of one of his 30-day billing cycles, his balance was $1360. He then
made a purchase for $470, so his balance jumped to $1830, and it remained
that amount for the next 10 days. Lowell then made a payment of $740, so
his balance for the last 10 days of the billing cycle was $1090. If his credit
card's APR is 28%, which of these expressions could be used to calculate the
amount Lowell was charged in interest for the billing cycle?
Answer: (0.28/365 •30)(620)
Step-by-step explanation:
which event most contributed to the changing troop levels shown in this graph? The Twenty-Sixth Amendment lowered the draft age to 18 from 21.
U.S. and North Vietnamese ships exchanged fire in the Gulf of Tonkin.
Congress expanded presidential powers to wage war under the War Powers Act.
Communist troops launched a series of attacks during the Tet Offensive.
The event that most contributed to the changing troop levels shown in the graph is when Communist troops launched a series of attacks during the Tet Offensive.
The Communist troops launched a series of attacks during the Tet Offensive to try to undermine American and South Vietnamese morale, cause a general uprising and seize control of the cities in South Vietnam.
However, this didn't go as planned, since the Communist troops suffered devastating losses on the battlefield.
The Tet Offensive, which was one of the most important turning points in the Vietnam War, led to changes in troop levels that are shown on the graph.
The Tet Offensive significantly increased troop levels because American forces had to respond with more soldiers and resources to defend against the attacks.
To know more about Tet Offensive visit:
https://brainly.in/question/16972839
#SPJ11
The event which most contributed to the changing troop levels shown in the graph was the Communist troops launching a series of attacks during the Tet Offensive.
The Tet Offensive was a series of attacks on the cities and towns of South Vietnam by the People's Army of Vietnam (PAVN) (also known as the North Vietnamese Army or NVA) and the National Liberation Front of South Vietnam (NLF), commonly known as the Viet Cong.
The Tet Offensive began in the early hours of 30th January 1968, during the Vietnam War. This event had a significant impact on public opinion and led to the escalation of the war.The graph in question, which depicts the troop levels, demonstrates that there was a considerable rise in US troop numbers during the years leading up to the Tet Offensive.
Following this event, troop numbers rose even higher before declining in the years that followed.
Therefore, the Communist troops launching a series of attacks during the Tet Offensive contributed most to the changing troop levels shown in the graph.
To know more about Tet Offensive, visit:
https://brainly.com/question/1114419
#SPJ11
Andrew and Kate have $30 to spend on dinner, tax, and gratuity at Mo’s Restaurant. Tax is 6%, and they will give a 15% tip on the total bill after taxes. Their dinner costs $21.00. Which statement correctly explains whether Andrew and Kate have enough money to pay their bill? '
4. A build-your-pasta restaurant charges a flat fee of $2.50 for pasta and $0.75 per ingredient. How much
does the pasta cost for 4 ingredients?
y =
I
The pasta plate will cost $
Answer:
Step-by-step explanation:
The pasta plate will cost $ 5.5
y=4x+2.5 ( x is the number of ingredients)
$ 2.5+(4*0.75)= 5.5 dollars
The following is a set of data from a sample of
n=5.
4 −9 −4 4 6
a. Compute the mean, median, and mode.
b. Compute the range, variance, standard deviation, and coefficient of variation.
c. Compute the Z scores. Are there any outliers?
d. Describe the shape of the data set.
a. The mean is -0.6, the median is 4, and there is no mode in the data set.
b. The range is 15, the variance is 35.2, the standard deviation is approximately 5.93, and the coefficient of variation is approximately -0.988.
c. The Z-scores for the data set are -0.68, -1.69, -0.68, -0.68, and 1.37. There are no outliers as none of the Z-scores exceed the threshold of ±3.
d. The shape of the data set is skewed to the left, indicating a negative skewness.
a. To calculate the mean, we sum up all the values and divide by the sample size:
Mean = (4 - 9 - 4 + 4 + 6) / 5 = -0.6
The median is the middle value when the data is arranged in ascending order:
Median = 4
The mode is the value that appears most frequently, but in this data set, none of the values are repeated, so there is no mode.
b. The range is calculated by finding the difference between the maximum and minimum values:
Range = Maximum value - Minimum value = 6 - (-9) = 15
The variance measures the average squared deviation from the mean:
Variance = ((4 - (-0.6))^2 + (-9 - (-0.6))^2 + (-4 - (-0.6))^2 + (4 - (-0.6))^2 + (6 - (-0.6))^2) / (5 - 1) = 35.2
The standard deviation is the square root of the variance:
Standard Deviation ≈ √35.2 ≈ 5.93
The coefficient of variation is the standard deviation divided by the mean, expressed as a percentage:
Coefficient of Variation ≈ (5.93 / 0.6) × 100 ≈ -0.988
c. The Z-score measures how many standard deviations a data point is away from the mean. To calculate the Z-scores, we subtract the mean from each data point and divide by the standard deviation:
Z1 = (4 - (-0.6)) / 5.93 ≈ -0.68
Z2 = (-9 - (-0.6)) / 5.93 ≈ -1.69
Z3 = (-4 - (-0.6)) / 5.93 ≈ -0.68
Z4 = (4 - (-0.6)) / 5.93 ≈ -0.68
Z5 = (6 - (-0.6)) / 5.93 ≈ 1.37
Since none of the Z-scores exceed the threshold of ±3, there are no outliers in the data set.
d. The shape of the data set can be determined by analyzing the skewness. A negative skewness indicates that the data is skewed to the left, which means that the tail of the distribution extends towards the lower values. In this case, the negative skewness suggests that the data set is skewed to the left.
To know more about mean , visit
https://brainly.com/question/1136789
#SPJ11
Which of the following verifies that g satisfies the hypotheses of the Mean Value Theorem on the interval [0, 8]
To verify that g satisfies the hypotheses of the Mean Value Theorem on the interval [0, 8], we need to ensure that two conditions are met:
1. Continuity: We need to check if g is continuous on the interval [0, 8]. This means that there should be no breaks, jumps, or holes in the graph of g over this interval. To verify continuity, we can examine if there are any vertical asymptotes, removable discontinuities, or jumps in the graph of g within the interval [0, 8].
2. Differentiability: We need to check if g is differentiable on the open interval (0, 8). This means that the derivative of g should exist and be finite at every point within the interval (0, 8). To verify differentiability, we can examine if the derivative of g exists and is finite for all values of x in the open interval (0, 8).
To summarize, to verify that g satisfies the hypotheses of the Mean Value Theorem on the interval [0, 8], we need to ensure that g is continuous on [0, 8] and differentiable on (0, 8). If both of these conditions are met, then g satisfies the hypotheses of the Mean Value Theorem on the interval [0, 8].
Mean Value and on the interval [0, 8] : https://brainly.com/question/19052862
#SPJ11
Simplify . (x ^ 5)/(x ^ 2).
Answer:
\(x {}^{3} \)
Step-by-step explanation:
\(\frac{(x {}^{5} )}{( {x}^{2} )}\)
\(x {}^{3} \)
four cubes of volumes $1 \text{ cm}^3$, $8 \text{ cm}^3$, $27 \text{ cm}^3$, and $125 \text{ cm}^3$ are glued together at their faces. what is the number of square centimeters in the smallest possible surface area of the resulting solid figure?
Answer: 194
Step-by-step explanation:
From the volumes, we deduce that the side lengths of the cubes are 1 cm, 2 cm, 3 cm, and 5 cm. We position the cubes as follows:
[asy]
unitsize(0.5 cm);
draw((0,0)--(5*dir(-30))--(5*dir(-30) + 5*dir(30))--(10*dir(-30))--(5*dir(-30) + 5*dir(-90))--(5*dir(-90))--(0,0));
draw((5*dir(-30))--(5*dir(-30) + 5*dir(-90)));
draw((0,0)--(0,2)--((0,2) + 2*dir(-30))--(2*dir(-30)));
draw((0,2)--((0,2) + 2*dir(30))--((0,2) + 2*dir(30) + 2*dir(-30))--(2*dir(30)));
draw((2*dir(-30))--(2*dir(-30) + dir(30))--(2*dir(-30) + dir(30) + (0,1))--(2*dir(-30) + 2*dir(30) + (0,1))--(2*dir(-30) + 2*dir(30) + (0,2)));
draw((2*dir(-30) + dir(30))--(3*dir(-30) + dir(30))--(3*dir(-30) + dir(30) + (0,1))--(2*dir(-30) + dir(30) + (0,1)));
draw((3*dir(-30) + dir(30) + (0,1))--(3*dir(-30) + 2*dir(30) + (0,1))--(2*dir(-30) + 2*dir(30) + (0,1)));
draw((2*dir(30) + (0,2))--(2*dir(30) + (0,3))--(2*dir(30) + 3*dir(-30) + (0,3))--(2*dir(30) + 3*dir(-30))--(dir(30) + 3*dir(-30)));
draw((2*dir(30) + (0,3))--(5*dir(30) + (0,3))--(5*dir(30) + 3*dir(-30) + (0,3))--(5*dir(30) + 3*dir(-30))--(5*dir(30) + 5*dir(-30)));
draw((3*dir(-30) + 2*dir(30))--(3*dir(-30) + 5*dir(30)));
draw((3*dir(-30) + 2*dir(30) + (0,3))--(3*dir(-30) + 5*dir(30) + (0,3)));
[/asy]
The surface area of a cube with side length $s$ is $6s^2$, so the total surface area of the cubes is $6 \cdot 1^2 + 6 \cdot 2^2 + 6 \cdot 3^2 + 6 \cdot 5^2 = 234$.
Note that every pair of cubes touches, and furthermore, they have maximum contact. (This is why this solid has the smallest possible area.) The area of contact of the 1-cube and the 2-cube is 1 square centimeter, so we must subtract this twice from 234 (because this portion of the area from both the 1-cube and 2-cube is not seen anymore).
Doing this for every pair of cubes, we find that the surface area of this solid is $234 - 2 \cdot 1^2 - 2 \cdot 1^2 - 2 \cdot 1^2 - 2 \cdot 2^2 - 2 \cdot 2^2 - 2 \cdot 3^2 = \boxed{194}$.
Consider the hypotheses shown below. Given that x
ˉ
=119,σ=27,n=46,α=0.10, complete parts a through c below. H 0
:μ=128
H A
⩽μ
=128
a. State the decision rule in terms of tho criteal value(s) of the test statistic: Reject the null hypothesis it the calculated value of the tost statistic, is otherwise, do not roject the null hypothesis. (Round to two decimal places as needed. Use a comma to separate answers as needed.) b. Stase the calculated value of the tost statistic. Tho best stasistic is (Round to toro decimal paces as needod.) c. State the conclusion. Beceuse the test statiski the null hypothesis and conclude the pepulation moan equal to 120 .
a. Decision rule: Reject the null hypothesis if the calculated z-value is less than or equal to -1.28. b. Calculated z-value: -1.8892. c. Conclusion: Reject the null hypothesis, indicating evidence that the population mean is less than 128.
To complete parts (a) through (c), we need to perform a hypothesis test for the given hypotheses
H0: μ = 128 (null hypothesis)
HA: μ ≤ 128 (alternative hypothesis)
Given: X= 119 (sample mean)
σ = 27 (population standard deviation)
n = 46 (sample size)
α = 0.10 (significance level)
a. The decision rule is to reject the null hypothesis if the calculated value of the test statistic is less than or equal to the critical value(s) of the test statistic. Since the alternative hypothesis is one-sided (μ ≤ 128), we will use a one-sample z-test and compare the calculated z-value with the critical z-value.
To find the critical z-value, we need to determine the z-value corresponding to the significance level α = 0.10. Looking up the critical value in the standard normal distribution table, we find that the critical z-value is -1.28 (rounded to two decimal places).
b. The calculated value of the test statistic, in this case, is the z-value. We can calculate the z-value using the formula
z = (X - μ) / (σ / √n)
Substituting the given values:
z = (119 - 128) / (27 / √46) ≈ -1.8892 (rounded to two decimal places)
c. The conclusion is based on comparing the calculated value of the test statistic with the critical value. Since the calculated z-value of -1.8892 is less than the critical z-value of -1.28, we have enough evidence to reject the null hypothesis. Therefore, we conclude that the population mean is less than 128.
The conclusion statement in part (c) is inconsistent with the given alternative hypothesis and should be revised accordingly.
To know more about null hypotheses:
brainly.com/question/28331914
#SPJ4
Please help!!
In the diagram, ΔMTD ≅ ΔGLS . Which statement is true?
∠M ≅ ∠G
DT ≅ LG
DM ≅ SL
∠T ≅ ∠S
Answer:
∠M ≅ ∠G
Step-by-step explanation:
Given:
ΔMTD ≅ ΔGLSThis means that triangle MTD is congruent to triangle GLS.
Two triangles are said to be congruent if:
all three corresponding sides are equal.all three corresponding angles are equal.Therefore:
∠M ≅ ∠G∠T ≅ ∠L∠D ≅ ∠SSimilarly:
TM ≅ LGDT ≅ SLDM ≅ SGTherefore, the true statement is ∠M ≅ ∠G.
The red function is a representation of y=f(x) which equation represents the blue function
Blue Graph has the point: (-2,2) and Red Graph has the point: (-2,0). Therefore, when you remove 2 units of y for blue graph you represent the red graph.
Answer Letter A
L 4.6.3 Test (CST): Linear Equations
me.
OA. y+4= -3(x-3)
OB. y-4=-3(x+3)
OC. y-4=3(x+3)
OD. y+4=3(x-3)
(3,-4)
The correct option is OA. y+4= -3(x-3). L 4.6.3 Test (CST): Linear Equations Solution: We are given that a line passes through (3,-4) and has a slope of -3.
We will use point slope form of line to obtain the equation of liney - y1 = m(x - x1).
Plugging in the values, we get,y - (-4) = -3(x - 3).
Simplifying the above expression, we get y + 4 = -3x + 9y = -3x + 9 - 4y = -3x + 5y = -3x + 5.
This equation is in slope intercept form of line where slope is -3 and y-intercept is 5.The above equation is not matching with any of the options given.
Let's try to put the equation in standard form of line,ax + by = c=> 3x + y = 5
Multiplying all the terms by -1,-3x - y = -5
We observe that option (A) satisfies the above equation of line, therefore correct option is OA. y+4= -3(x-3).
Thus, the correct option is OA. y+4= -3(x-3).
For more question on equation
https://brainly.com/question/17145398
#SPJ8
A cab company charges $5 per cab ride, plus an additional $1 per mile driven. How long is a cab ride that costs $14?
Answer:
The answer is 9 miles.
Step-by-step explanation:
14-5=9.
Answer:
9 miles
Step-by-step explanation:
So if you pay $5 for the cab ride then 1$ for every mile driven and you have to pay a total of $14 you go 14-5=9
i dont get it
help please :)
Question 37 1 pts Which of the following is a solution to the differential equation: t². d^2y/dt^2 - 6t dy,dt + 12 = 0
a.y=t² + 1 b.y=t³+2t^4
c.no solution d.y = t-1 Question 38 1 pts Newton's law of cooliing states that the rate of change of the temperature T of an object is proportional to the temperature difference between the temperature S of the surroundings and the temperature T. Write down the differential equation. A cup of tea is prepared from boiling water at 100 degrees and cools to 50 degrees in 3 minutes. The temperature in the room is 20 degrees. What will the tea temperature be after a very long time? a.dT/dt=k(S-T); T≈ 20 degrees after a very long time b.dt/dT =k(ST); T≈ 30 degrees after a very long time c.dt/dT =K(S-T); T≈ 0 degrees after a very long time d.dt/dT = K (S+T); T≈ 30 degrees after a very long time
Question 37: The following is a solution to the differential equation y = t - 1. d.
Question 38: The tea temperature be after a very long time is dT/dt = k(S - T); T ≈ 20 degrees after a very long time. a.
To determine the solution to the given differential equation: t²(d²y/dt²) - 6t(dy/dt) + 12 = 0, we can solve the equation by assuming a solution in the form of y = tⁿ, where n is a constant.
By differentiating y with respect to t, we can substitute the derivatives into the differential equation to determine the value of n.
Let's differentiate y = tⁿ with respect to t:
dy/dt = n × tⁿ⁻¹
d²y/dt² = n(n-1) × tⁿ⁻²
Substituting these derivatives into the differential equation:
t²(n(n-1)tⁿ⁻²) - 6t(ntⁿ⁻¹) + 12 = 0
Simplifying the equation:
n(n-1)tⁿ - 6ntⁿ + 12 = 0
n(n-1)tⁿ - 6ntⁿ = -12
Since this equation must hold for all values of t, the coefficients of the tⁿ terms on both sides of the equation must be equal.
We can equate the coefficients:
n(n-1) = 0
n = 0 or n = 1
The general solution to the differential equation is y = c₁ + c₂ × t, where c₁ and c₂ are constants.
According to Newton's law of cooling, the rate of change of the temperature T of an object is proportional to the temperature difference between the object's temperature T and the surroundings' temperature S.
We can write the differential equation as follows:
dT/dt = k(S - T)
dT/dt represents the rate of change of temperature, k is the proportionality constant, and (S - T) represents the temperature difference between the object and the surroundings.
The cup of tea starts at 100 degrees and cools to 50 degrees in 3 minutes, with the room temperature at 20 degrees.
We can assume that the temperature of the tea will tend toward the room temperature as time goes to infinity.
This option correctly represents that as time goes to infinity, the temperature T of the tea will approach the temperature S of the surroundings, which is approximately 20 degrees.
For similar questions on differential equation
https://brainly.com/question/1164377
#SPJ8
if s is a linearly dependent set, then each vector is a linear combination of the other vectors in s.
a. True
b. False
The statement "if s is a linearly dependent set, then each vector is a linear combination of the other vectors in s" is false.
What is the linearly dependent set?A vector set's linear independence is an important attribute. If no vector in a set can be written as a linear combination of the other vectors in the set, the set is said to be linearly independent. The set is said to be linearly dependent if any of the vectors can be represented as a linear combination of the others.
If S is a linearly dependent set, then each vector in S is a linear combination of the others.
This is false.
For example, v₁ = (1,0), v₂ = (2,0), and v₃ = (1,1). Then v₂ = 2v₁ but v₃ is not a linear combination of v₁ and v₂, since it is not a multiple of v₁. But 2v₁ - 1v₂ + 0 v₃ = 0.
If an indexed set of vector S is linear dependent, then it is only necessary that one of the vectors is in the set.
Hence, the correct answer would be option (B).
To learn more about linearly dependent set here:
https://brainly.com/question/13872353
#SPJ12
The length of each side of equilateral triangle T is 6 times the length of each side of equilateral triangle X.
Quantity A The ratio of the length of one side of T to
the length of another side of
Quantity B The ratio of the length of one side of X to
the length of another side of X
•A. Quantity A is greater.
• B. Quantity B is greater.
C. The two quantities are equal.
• D. The relationship cannot be determined from the information given.
C. The two quantities are equal. We can see that both Quantity A and Quantity B have the same value of 1. Hence, the two quantities are equal.
Since we know that the length of each side of triangle T is 6 times the length of each side of triangle X, we can set up the following equations:
Length of side of T = 6 * Length of side of X
To compare the ratios of the sides, we can use the following formula:
Ratio of one side to another = Length of one side / Length of another side
For Quantity A, we can choose any two sides of triangle T and find their ratio. Let's choose the two adjacent sides:
To further explain the concept, let's consider the properties of equilateral triangles.
An equilateral triangle is a special type of triangle where all three sides are equal in length. Hence, if we know the length of one side, we automatically know the length of all three sides.
Now, coming back to the given question, we are told that the length of each side of triangle T is 6 times the length of each side of triangle X. This means that if the length of one side of triangle X is 'a', then the length of one side of triangle T is 6 times 'a', which is '6a'.
Hence, we can write:
Length of side of T = 6a
Length of side of X = a
Now, let's compare the ratios of the sides.
For Quantity A, we need to find the ratio of one side of triangle T to another side of T. Let's choose the two adjacent sides:
Ratio of adjacent sides of T = Length of side of T / Length of adjacent side of T
Since all three sides of an equilateral triangle are equal, the adjacent side of T is also 6a. Hence, we get:
Ratio of adjacent sides of T = (6a) / (6a) = 1
For Quantity B, we need to find the ratio of one side of triangle X to another side of X. Let's again choose the two adjacent sides:
Ratio of adjacent sides of X = Length of side of X / Length of adjacent side of X
Since all three sides of an equilateral triangle are equal, the adjacent side of X is also 'a'.
To know more about quantities visit :-
https://brainly.com/question/14581760
#SPJ11
can someone help me with this please?
Answer:
download the app photomath and right your solution
Estimate how many times you would have to fold a sheet of paper until it becomes as thick as a large dictionary (approximately 10 cm thick). 1000 times 100 times 500 times 50 times 10 times
Based on the calculations, none of the given options result in a thickness that matches or exceeds the thickness of a large dictionary. Therefore, none of the options provided are correct.
To estimate how many times you would have to fold a sheet of paper until it becomes as thick as a large dictionary, we need to consider the concept of exponential growth in folding.
Each time you fold a sheet of paper in half, its thickness doubles. So, if we denote the initial thickness of the paper as 1 fold, then after the first fold it becomes 2 folds thick, after the second fold it becomes 4 folds thick, and so on.
Given that a large dictionary is approximately 10 cm thick, we need to find the number of folds that would result in a thickness of 10 cm.
Let's calculate the number of folds required for each given option:
1000 times:
Starting with 1 fold, after 1000 folds the thickness would be 2^1000 folds, which is an extremely large number. It would far exceed the thickness of a large dictionary, so this option is not correct.
100 times:
Starting with 1 fold, after 100 folds the thickness would be 2^100 folds. Although this number is large, it is still far less than the thickness of a large dictionary. So this option is not correct either.
500 times:
Starting with 1 fold, after 500 folds the thickness would be 2^500 folds. This is also an extremely large number that surpasses the thickness of a large dictionary, so this option is not correct.
50 times:
Starting with 1 fold, after 50 folds the thickness would be 2^50 folds. While this is a large number, it is still significantly less than the thickness of a large dictionary. So this option is not correct.
10 times:
Starting with 1 fold, after 10 folds the thickness would be 2^10 folds, which equals 1024 folds. This is still less than the thickness of a large dictionary, so this option is not correct.
To know more about thickness,
https://brainly.com/question/30613346
#SPJ11
3. If a 6-pack of soda costs 6 dollars, what is the cost of each soda? What is 1 point
the unit rate?
$0
$1
$3
$6
What are all the expressions equivalent to 2(x+3)
Answer:
2 x + 6 and x = -3
Step-by-step explanation:
Find the value of p : p/3=4
Answer:
p = 12
Step-by-step explanation:
p ÷ 3 = 4
p = 12
1. If f(x) = (3x-2)/(2x+3), then f'(x) =
Answer:
\(f'(x)= \frac{13}{(2x+3)^2}\\\)
Step-by-step explanation:
\(f(x)= \frac{3x-2}{2x+3} \\\)
\(f'(x)=\frac{dy}{dx} = \frac{d}{dx}(\frac{3x-2}{2x+3})\\ f'(x)= \frac{(2x+3)\frac{d}{dx}(3x-2)-(3x-2)\frac{d}{dx}(2x+3) }{(2x+3)^{2} } \\f'(x)= \frac{(2x+3)(3)-(3x-2)(2)}{(2x+3)^{2} } \\\)
\(f'(x)= \frac{6x+9-6x+4}{(2x+3)^{2} }\\ f'(x)= \frac{13}{(2x+3)^2}\\\)